The Student Room Group

STEP Maths I, II, III 1989 solutions

Scroll to see replies

Reply 1
I am no expert at STEP, so don't hesitate to correct me, it is more than likely made that I have made a mistake. :redface:

STEP I - Question 8

Using good old de Moivre's theorem;

cos(4θ)=(cos(θ)+isin(θ))4=(cos4(θ)+4icos3(θ)sin(θ)6cos2(θ)sin2(θ)4isin3(θ)cos(θ)+sin4(θ))=cos4(θ)6cos2(θ)sin2(θ)=8cos4(θ)8cos2(θ)+1\cos(4\theta) = \Re( \cos(\theta) + i\sin(\theta))^{4} = \Re(\cos^{4}(\theta) + 4i\cos^{3}(\theta)\sin(\theta) - 6\cos^{2}(\theta)\sin^{2}(\theta) - 4i\sin^{3}(\theta)\cos(\theta) + \sin^{4}(\theta) ) = cos^{4}(\theta) - 6\cos^{2}(\theta)\sin^{2}(\theta) = 8\cos^{4}(\theta) - 8\cos^{2}(\theta) + 1


(using cos2(θ)+sin2(θ)1 \cos^{2}(\theta) + \sin^{2}(\theta) \equiv 1

and similary;

[br]cos(6θ)=(cos(θ)+isin(θ))6=(cos6(θ)+6icos5(θ)sin(θ)15cos4(θ)sin2(θ)20isin3(θ)cos3(θ)+15sin4(θ)cos2(θ)+6icos(θ)sin5(θ)sin6(θ))=32cos6(θ)48cos4(θ)+18cos2(θ)1[br]\cos(6\theta) = \Re( \cos(\theta) + i\sin(\theta))^{6} = \Re(\cos^{6}(\theta) + 6i\cos^{5}(\theta)\sin(\theta) - 15\cos^{4}(\theta)\sin^{2}(\theta) - 20i\sin^{3}(\theta)\cos^{3}(\theta) + 15\sin^{4}(\theta)\cos^{2}(\theta) + 6i\cos(\theta)\sin^{5}(\theta) - \sin^{6}(\theta) ) = 32\cos^{6}(\theta) - 48\cos^{4}(\theta) +18\cos^{2}(\theta) - 1

Now, consider: 12cos(6θ)12cos(4θ) \frac{1}{2}\cos(6\theta) - \frac{1}{2}cos(4\theta)

12cos(6θ)12cos(4θ)=16c628c4+13c21 \frac{1}{2}\cos(6\theta) - \frac{1}{2}cos(4\theta) = 16c^{6} - 28c^{4} + 13c^{2} -1

Where c is cos(θ) \cos(\theta) .

Now let x=cos(θ)x = cos(\theta),

so:

16x628x4+13x21=16c628c4+13c21=12cos(6θ)12cos(4θ)=0[br] 16x^{6} - 28x^{4} + 13x^{2} -1 = 16c^{6} - 28c^{4} + 13c^{2} -1 = \frac{1}{2}\cos(6\theta) - \frac{1}{2}cos(4\theta) = 0[br]

cos(6θ)=cos(4θ)\Rightarrow \cos(6\theta) = \cos(4\theta)

6θ=2nπ±4θ \Rightarrow 6\theta = 2n\pi \pm 4\theta

Which gives;

θ=nπ5 \theta = \frac{n\pi}{5} (n=1,2,3,4 n= 1,2,3,4 ).
These values of θ\theta give distinct values of cos(θ)\cos(\theta)

or

θ=nπ \theta = n\pi (n=0,1 n= 0,1 )


Hence the roots of the equation are:

x=cos(0),cos(π),cos(π5),cos(2π5),cos(3π5) x = \cos(0), \cos(\pi), \cos(\frac{\pi}{5}), \cos(\frac{2\pi}{5}), \cos(\frac{3\pi}{5}) and cos(4π5) \cos(\frac{4\pi}{5}) .
Reply 2
STEP III - Question 10

Lets assume that the result is true for n = k;

r=1kr(r+1)(r+2)(r+3)(r+4)=16k(k+1)(k+2)(k+3)(k+4)(k+5)\displaystyle\sum_{r=1}^k r(r+1)(r+2)(r+3)(r+4) = \frac{1}{6}k(k+1)(k+2)(k+3)(k+4)(k+5) (*)

[br]r=1k+1r(r+1)(r+2)(r+3)(r+4)=16k(k+1)(k+2)(k+3)(k+4)(k+5)+(k+1)(k+2)(k+3)(k+4)(k+5)[br]\Rightarrow \displaystyle\sum_{r=1}^{k+1} r(r+1)(r+2)(r+3)(r+4) = \frac{1}{6}k(k+1)(k+2)(k+3)(k+4)(k+5) + (k+1)(k+2)(k+3)(k+4)(k+5)

r=1k+1r(r+1)(r+2)(r+3)(r+4)=16(k+1)(k+2)(k+3)(k+4)(k+5)(k+6)\Rightarrow \displaystyle\sum_{r=1}^{k+1} r(r+1)(r+2)(r+3)(r+4) = \frac{1}{6}(k+1)(k+2)(k+3)(k+4)(k+5)(k+6)

This is the same as (*) except k+1 replaces k. Hence if the result is true for n=k, its true for n= k+1.

When n=1,

LHS of (*) = 1x2x3x4x5 = 120
RHS of (*) = (1/6)x1x2x3x4x5x6 = 120.

So (*) is true for n=1.
Hence, by induction;

r=1nr(r+1)(r+2)(r+3)(r+4)=16n(n+1)(n+2)(n+3)(n+4)(n+5)\displaystyle\sum_{r=1}^n r(r+1)(r+2)(r+3)(r+4) = \frac{1}{6}n(n+1)(n+2)(n+3)(n+4)(n+5) n1 \forall n \geq 1 .


Since;

r5<r(r+1)(r+2)(r+3)(r+4) r^{5} < r(r+1)(r+2)(r+3)(r+4)

r=1nr5<r=1nr(r+1)(r+2)(r+3)(r+4) \Rightarrow \displaystyle\sum_{r=1}^n r^{5} < \displaystyle\sum_{r=1}^n r(r+1)(r+2)(r+3)(r+4)

r=1nr5<16n(n+1)(n+2)(n+3)(n+4)(n+5) \Rightarrow \displaystyle\sum_{r=1}^n r^{5} < \frac{1}{6}n(n+1)(n+2)(n+3)(n+4)(n+5).

Using (*);

r=0n1r(r1)(r2)(r3)(r4)=16n(n1)(n2)(n3)(n4)(n5)\displaystyle\sum_{r=0}^{n-1} r(r-1)(r-2)(r-3)(r-4) = \frac{1}{6}n(n-1)(n-2)(n-3)(n-4)(n-5)


r(r1)(r2)(r3)(r4)<r5 r(r-1)(r-2)(r-3)(r-4) < r^{5}

r=0n1r5>r=0n1r(r1)(r2)(r3)(r4) \Rightarrow \displaystyle\sum_{r=0}^{n-1} r^{5} > \displaystyle\sum_{r=0}^{n-1} r(r-1)(r-2)(r-3)(r-4)

r=0n1r5>16(n5)(n4)(n3)(n2)(n1)n \Rightarrow \displaystyle\sum_{r=0}^{n-1} r^{5} > \frac{1}{6}(n-5)(n-4)(n-3)(n-2)(n-1)n


In this case, f(x)=x5f(x) = x^{5}

From the previous parts, we know that:

r=0n1a6n6r5>a66n6(n5)(n4)(n3)(n2)(n1)n\displaystyle\sum_{r=0}^{n-1} \frac{a^{6}}{n^{6}}r^{5} > \frac{a^{6}}{6n^{6}}(n-5)(n-4)(n-3)(n-2)(n-1)n

and clearly, also:

r=0n1a6n6r5<a66n6n(n+1)(n+2)(n+3)(n+4)(n+5) \displaystyle\sum_{r=0}^{n-1} \frac{a^{6}}{n^{6}}r^{5} < \frac{a^{6}}{6n^{6}}n(n+1)(n+2)(n+3)(n+4)(n+5)

as nn\longrightarrow \infty ,

a66n6(n5)(n4)(n3)(n2)(n1)n \frac{a^{6}}{6n^{6}}(n-5)(n-4)(n-3)(n-2)(n-1)n and a66n6n(n+1)(n+2)(n+3)(n+4)(n+5)a66\frac{a^{6}}{6n^{6}}n(n+1)(n+2)(n+3)(n+4)(n+5) \longrightarrow \frac{a^{6}}{6}

So,

limnr=0n1a6n6r5=a66\lim_{n\to \infty} \displaystyle\sum_{r=0}^{n-1} \frac{a^{6}}{n^{6}}r^{5} = \frac{a^{6}}{6}

Using a similar arguement, we can show that:

limnr=1na6n6r5=a66\lim_{n\to \infty} \displaystyle\sum_{r=1}^{n} \frac{a^{6}}{n^{6}}r^{5} = \frac{a^{6}}{6}

We have shown that the limits exist and are equal to a66\frac{a^{6}}{6}.

Hence;


0ax5dx=a66\displaystyle\int^a_0 x^{5} \, \mathrm{d}x = \frac{a^{6}}{6}
Reply 3
STEP I, Q2

The For x>0x > 0, find xlnxdx\displaystyle\int x\ln x dx

Using integration by parts (uvdx=uvuvdx\displaystyle\int u'v dx = uv - \int uv' dx ):

xlnxdx=12x2lnx12x21xdx\displaystyle\int x\ln x dx = \frac{1}{2}x^2 \cdot \ln x - \int \frac{1}{2}x^2 \cdot \frac{1}{x} dx

=12(x2lnxxdx)\displaystyle = \frac{1}{2}(x^2 \cdot \ln x - \int x dx)

=12(x2lnx12x2)+C\displaystyle = \frac{1}{2}(x^2 \cdot \ln x - \frac{1}{2} x^2) + C

=14x2(2lnx1)+C\displaystyle = \frac{1}{4}x^2(2 \ln x - 1) + C



By approximating the area corresponding to 01xln(1/x)dx\int^1_0 x\ln(1/x) dx by n rectangles of equal width and with their top right hand vertices on the curve y=xln(1/x)y = x\ln(1/x), show that, as nn \to \infty,

12(1+1n)lnn1n2[ln(n!0!)+ln(n!1!)+ln(n!2!)+...ln(n!(n1)!)]14\displaystyle\frac{1}{2}(1 + \frac{1}{n})\ln n - \frac{1}{n^2}[ln(\frac{n!}{0!}) + ln(\frac{n!}{1!}) + ln(\frac{n!}{2!}) + ... ln(\frac{n!}{(n-1)!})] \to \frac{1}{4}



()(*) Firstly, note that

01xln(1/x)dx\displaystyle \int^1_0 x\ln(1/x) dx

=01x(ln(x))dx\displaystyle= \int^1_0 x(-\ln(x)) dx

=01xln(x)dx\displaystyle= -\int^1_0 x\ln(x) dx

=[14x2(2lnx1)]01\displaystyle= -[\frac{1}{4}x^2(2 \ln x - 1)]^1_0 (by the above indefinite integral)

=limt0[14x2(2lnx1)]t1\displaystyle= \lim_{t \to 0} -[\frac{1}{4}x^2(2 \ln x - 1)]^1_t

=limt014(12(ln11)t2(2lnt1))\displaystyle= \lim_{t \to 0} -\frac{1}{4}(1^2(\ln 1 - 1) - t^2(2 \ln t - 1))

=limt014(1+2t2lntt2))\displaystyle= \lim_{t \to 0} \frac{1}{4}(1 + 2t^2\ln t - t^2))

=limt014(1+2t(tlnt)t2))\displaystyle= \lim_{t \to 0} \frac{1}{4}(1 + 2t(t\ln t) - t^2))

=14\displaystyle= \frac{1}{4}



The sum of the areas of our rectangles is going to be

()r=1n1n(rnln(1r/n))\displaystyle (**) \sum^n_{r=1} \frac{1}{n} (\frac{r}{n}\ln(\frac{1}{r/n}))

=r=1n1n(rnln(nr))\displaystyle = \sum^n_{r=1} \frac{1}{n} (\frac{r}{n}\ln(\frac{n}{r}))

=1n2r=1nrln(nr)\displaystyle = \frac{1}{n^2} \sum^n_{r=1} r\ln(\frac{n}{r})

=1n2(1ln(n1)+2ln(n2)+3ln(n3)+...+nln(nn))\displaystyle = \frac{1}{n^2} (1\ln(\frac{n}{1}) + 2\ln(\frac{n}{2}) + 3\ln(\frac{n}{3}) + ... + n\ln(\frac{n}{n}))

=1n2ln((n1)(n2)2(n3)3(nn)n)\displaystyle = \frac{1}{n^2} \ln((\frac{n}{1})(\frac{n}{2})^2(\frac{n}{3})^3 \cdot \cdot \cdot (\frac{n}{n})^n)

=1n2ln(nn2n3nn12233nn)\displaystyle = \frac{1}{n^2} \ln(\frac{n\cdot n^2\cdot n^3\cdot\cdot\cdot n^n}{1\cdot 2^2\cdot 3^3\cdot\cdot\cdot n^n})

=1n2ln(n1+2+3+4+...+n12233nn)\displaystyle = \frac{1}{n^2} \ln(\frac{n^{1+2+3+4+...+n}}{1\cdot 2^2\cdot 3^3\cdot\cdot\cdot n^n})

=1n2(ln(n1+2+3+4+...+n)ln(12233nn))\displaystyle = \frac{1}{n^2} (ln(n^{1+2+3+4+...+n})-\ln(1\cdot 2^2\cdot 3^3\cdot\cdot\cdot n^n))

=1n2(ln(n12n(n+1))ln(12233nn))\displaystyle = \frac{1}{n^2} (ln(n^{\frac{1}{2}n(n+1)})-\ln(1\cdot 2^2\cdot 3^3\cdot\cdot\cdot n^n))

=1n2(12n(n+1)ln(n)ln(12233nn))\displaystyle = \frac{1}{n^2} ({\frac{1}{2}n(n+1)}ln(n)-\ln(1\cdot 2^2\cdot 3^3\cdot\cdot\cdot n^n))

=1n212n(n+1)ln(n)1n2ln(12233nn)\displaystyle = \frac{1}{n^2}{\frac{1}{2}n(n+1)}ln(n) - \frac{1}{n^2} \ln(1\cdot 2^2\cdot 3^3\cdot\cdot\cdot n^n)

=12(1+1n)ln(n)1n2ln(12233nn)\displaystyle = \frac{1}{2}(1+\frac{1}{n})ln(n) - \frac{1}{n^2} \ln(1\cdot 2^2\cdot 3^3\cdot\cdot\cdot n^n)



Claim: 12233kk=k!0!k!1!k!2!k!k!1\cdot 2^2\cdot 3^3\cdot\cdot\cdot k^k = \frac{k!}{0!}\cdot\frac{k!}{1!}\cdot\frac{k!}{2!} \cdot\cdot\cdot\frac{k!}{k!}

So for k + 1, we must prove that k!0!k!1!k!2!(k+1)!(k+1)!\frac{k!}{0!}\cdot\frac{k!}{1!}\cdot\frac{k!}{2!} \cdot \cdot \cdot \frac{(k+1)!}{(k+1)!} is the equation we arrive at.

Basis case (let k=1): 11=1!0!1!1!=11^1 = \frac{1!}{0!}\cdot\frac{1!}{1!} = 1

Inductive step: 12233kk(k+1)k+1=(k+1)k+1(k!0!k!1!k!2!k!k!)1\cdot 2^2 \cdot 3^3 \cdot \cdot \cdot k^k\cdot (k+1)^{k+1} = (k+1)^{k+1}(\frac{k!}{0!} \cdot \frac {k!}{1!} \cdot \frac{k!}{2!} \cdot \cdot \cdot \frac{k!}{k!})

=((k+1)k!0!(k+1)k!1!(k+1)k!2!(k+1)k!k!) = ((k+1)\frac{k!}{0!}(k+1) \cdot \frac{k!}{1!}(k+1) \cdot \frac{k!}{2!} \cdot\cdot\cdot (k+1)\frac{k!}{k!})

=((k+1)!0!(k+1)!1!(k+1)!2!(k+1)!k!) = (\frac{(k+1)!}{0!}\cdot\frac{(k+1)!}{1!}\cdot\frac{(k+1)!}{2!}\cdot\cdot\cdot\frac{(k+1)!}{k!})

=((k+1)!0!(k+1)!1!(k+1)!2!(k+1)!k!(k+1)!(k+1)!) = (\frac{(k+1)!}{0!}\cdot\frac{(k+1)!}{1!}\cdot\frac{(k+1)!}{2!}\cdot\cdot\cdot\frac{(k+1)!}{k!}\cdot \frac{(k+1)!}{(k+1)!})



So,

()=12(1+1n)ln(n)1n2ln(n!0!n!1!n!2!n!n!)\displaystyle (**) = \frac{1}{2}(1+\frac{1}{n})ln(n) - \frac{1}{n^2} \ln(\frac{n!}{0!}\cdot\frac{n!}{1!}\cdot\frac{n!}{2!}\cdot\cdot\cdot\frac{n!}{n!})

=12(1+1n)ln(n)1n2(ln(n!0!)+ln(n!1!)+ln(n!2!)+...+ln(n!(n1)!)+ln(n!n!))\displaystyle = \frac{1}{2}(1+\frac{1}{n})ln(n) - \frac{1}{n^2} (\ln(\frac{n!}{0!}) + \ln(\frac{n!}{1!}) + \ln(\frac{n!}{2!}) + ... + \ln(\frac{n!}{(n-1)!}) + \ln(\frac{n!}{n!}))

=12(1+1n)ln(n)1n2(ln(n!0!)+ln(n!1!)+ln(n!2!)+...+ln(n!(n1)!))\displaystyle = \frac{1}{2}(1+\frac{1}{n})ln(n) - \frac{1}{n^2} (\ln(\frac{n!}{0!}) + \ln(\frac{n!}{1!}) + \ln(\frac{n!}{2!}) + ... + \ln(\frac{n!}{(n-1)!}))


As nn \to \infty, our sum tends to the integral ()(*). So,

12(1+1n)ln(n)1n2(ln(n!0!)+ln(n!1!)+ln(n!2!)+...+ln(n!(n1)!))14\displaystyle \frac{1}{2}(1+\frac{1}{n})ln(n) - \frac{1}{n^2} (\ln(\frac{n!}{0!}) + \ln(\frac{n!}{1!}) + \ln(\frac{n!}{2!}) + ... + \ln(\frac{n!}{(n-1)!})) \to \frac{1}{4}


is this ok? have i missed anything out or not explained each step in enough detail?
Reply 4
Question 4, STEP I

With six points, each point is attached to five other points, so at least three lines radiating from A must be of the same colour (gold, say). The points which are joined to A must then be joined to each other by silver lines, or an entirely gold triangle would be made. However, then a triangle with entirely silver edges can be made from these three points.

Diagram showing five points is attached.
Reply 5
Question 5, STEP I

(1+x)n=(n0)+(n1)x+(n2)x2++(nn)xn\displaystyle (1+x)^{n} = \binom{n}{0} + \binom{n}{1}x + \binom{n}{2}x^{2} + \cdots + \binom{n}{n}x^{n}

a)
Let x = 1.

2n=(n0)+(n1)+(n2)++(nn)\displaystyle 2^{n} = \binom{n}{0} + \binom{n}{1} + \binom{n}{2} + \cdots + \binom{n}{n}

Let x = -1

0=(n0)(n1)+(n2)+(nn)\displaystyle 0 = \binom{n}{0} - \binom{n}{1} + \binom{n}{2} - \cdots + \binom{n}{n}

As n is even. So

(n0)+(n2)++(nn)=(n1)+(n3)++(nn1)=2n1\displaystyle \binom{n}{0} + \binom{n}{2} + \cdots + \binom{n}{n} = \binom{n}{1} + \binom{n}{3} + \cdots + \binom{n}{n-1} = 2^{n-1}

As their sum is 2n\displaystyle 2^{n}

b) Suppose that

(k1)+2(k2)++k(kk)=k2k1\displaystyle \binom{k}{1} + 2\binom{k}{2} + \cdots + k\binom{k}{k} = k2^{k-1}

Then (k0)+2(k1)+3(k2)++(k+1)(kk)=k2k1+2k\displaystyle \binom{k}{0} + 2\binom{k}{1} + 3\binom{k}{2} + \cdots + (k+1)\binom{k}{k} = k2^{k-1} + 2^{k}

Note that (kr)+(kr1)=(k+1r)\displaystyle \binom{k}{r} + \binom{k}{r-1} = \binom{k+1}{r} and that (kk)=(k+1k+1)\displaystyle \binom{k}{k} = \binom{k+1}{k+1}

So, upon adding these two expressions, we get

(k+11)+2(k+12)++(k+1)(k+1k+1)=k2k1+k2k1+2k=(k+1)2k\displaystyle \binom{k+1}{1} + 2\binom{k+1}{2} + \cdots + (k+1)\binom{k+1}{k+1} = k2^{k-1} + k2^{k-1} + 2^{k} = (k+1)2^{k}

Also, (11)=1=1×20\displaystyle \binom{1}{1} = 1 = 1 \times 2^{0}

So true by induction.

r=0n(r+(1)r)(nr)=((n1)+2(n2)++n(nn))+((n0)(n1)++(1)n(nn))\displaystyle \displaystyle \sum_{r=0}^{n} \left(r + (-1)^{r}\right) \binom{n}{r} = \left(\binom{n}{1} + 2\binom{n}{2} + \cdots + n \binom{n}{n}\right) + \left(\binom{n}{0} - \binom{n}{1} + \cdots + (-1)^{n}\binom{n}{n}\right)

=n2n1\displaystyle = n2^{n-1}
Reply 6
STEP II Q1

[br]cos(3θ)=cos(2θ+θ)=cos(2θ)cos(θ)sin(2θ)sin(θ)=(2cos2(θ)1)cos(θ)2sin(θ)cos(θ)sin(θ)=2cos3(θ)cos(θ)2(1cos2(θ))cos(θ)=4cos3(θ)3cos(θ).[br][br]\cos(3\theta) = \cos(2\theta + \theta) = \cos(2\theta)\cos(\theta) - \sin(2\theta)\sin(\theta) = (2cos^{2}(\theta) - 1)\cos(\theta) - 2\sin(\theta)\cos(\theta)sin(\theta) = 2cos^{3}(\theta) - \cos(\theta) - 2(1-\cos^{2}(\theta))\cos(\theta) = 4cos^{3}(\theta) - 3\cos(\theta).[br]

Substituting x= y-a into the equation:

24x372x2+66x19=24(ya)372(ya)2+66(ya)19=0 24x^{3} - 72x^{2} + 66x -19 = 24(y-a)^{3} - 72(y-a)^{2} + 66(y-a) -19 = 0

Notice that the reduced form shown in the question has no z2z^{2} term, so we need to find the value of a which will get rid of the y2y^{2} term.

Expanding out and collectiing terms we get:

24y372(a+1)y2+(72a2+144a+66)y24a372a266a19=0 24y^{3} - 72(a+1)y^{2} + (72a^{2} + 144a +66)y - 24a^{3} - 72a^{2} -66a -19 = 0

So the value of a to get rid of the y2y^{2} is -1.

So we now have:

24y372(a+1)y2+(72a2+144a+66)y24a372a266a19=24t36y1=0 24y^{3} - 72(a+1)y^{2} + (72a^{2} + 144a +66)y - 24a^{3} - 72a^{2} -66a -19 = 24t^{3} - 6y -1 = 0

24y36y=1 \Rightarrow 24y^{3} - 6y = 1 (*)

Substituting y=z/b into (*);

24(zb)36(zb)=124(\frac{z}{b})^{3} - 6(\frac{z}{b}) = 1

dividing both sides by 2 and multiplying by bb;

12z3b23z=b212\frac{z^{3}}{b^{2}} - 3z = \frac{b}{2} .

And so to make this equal to;

4z33z=b24z^{3}- 3z = \frac{b}{2}

we make b = ±3\pm \sqrt3.

Lets take b = 3\sqrt3 and let z = cos(θ)\cos(\theta)

so;

4z33z=4cos3(θ)3cos(θ)=cos(3θ)=32 4z^{3}- 3z = 4\cos^{3}(\theta) - 3\cos(\theta) = \cos(3\theta) = \frac{\sqrt3}{2}

3θ=pi6,11π6\Rightarrow 3\theta = \frac{pi}{6}, \frac{11\pi}{6} and 13π6 \frac{13\pi}{6} (these give distinct values of cosθcos\theta, and hence, three distinct solutions.)


we know that; x = y - a = (z/b) - a.

Hence the solutions of the equation are:

x=cos(π18)3+1,cos(11π18)3+1 x = \frac{\cos(\frac{\pi}{18})}{\sqrt3} + 1 , \frac{\cos(\frac{11\pi}{18})}{\sqrt3} +1 and cos(13π18)3+1 \frac{\cos(\frac{13\pi}{18})}{\sqrt3} + 1
Question 11, STEP I.

Vertical component = vsinθv \sin \theta
Using s = ut + 1/2at^2.
h=vsinθ1/2gt2-h = v \sin \theta - 1/2gt^2
1/2gt2vsinθh=01/2gt^2 - v \sin \theta - h = 0
Quadratic in t. Ignoring the negative root.
t=vsinθ+v2sin2θ+2hggt = \dfrac{v \sin \theta + \sqrt{v^2 \sin^2 \theta + 2hg}}{g}
R = horizontal component x time.
Horizontal component = vcosθv \cos \theta
R=vcosθ(vsinθ+v2sin2θ+2hggR = v \cos \theta (\dfrac{v \sin \theta + \sqrt{v^2 \sin^2 \theta + 2hg}}{g}
R=v2sinθcosθ+vcosθv2sin2θ+2hggR = \dfrac{v^2 \sin \theta \cos \theta + v \cos \theta \sqrt{v^2 \sin^2 \theta + 2hg}}{g}
Multiply both numerator and denominator by 2.
R=2v2sinθcosθ+2vcosθv2sin2θ+2hg2gR = \dfrac{2v^2 \sin \theta \cos \theta + 2v \cos \theta \sqrt{v^2 \sin^2 \theta + 2hg}}{2g}
Using sin(2θ)=2sinθcosθ\sin (2\theta) = 2 \sin \theta \cos \theta
We can simplify it.
R=v2sin2θ+2vcosθv2sin2θ+2hg2gR = \dfrac{v^2 \sin 2\theta + 2v \cos \theta \sqrt{v^2 \sin^2 \theta + 2hg}}{2g}
Splitting up Mr. Big Fraction.
R=v2sin2θ2g+2vcosθv2sin2θ+2hg2gR = \dfrac{v^2 \sin 2\theta}{2g} + \dfrac{2v \cos \theta \sqrt{v^2 \sin^2 \theta + 2hg}}{2g}

Ignore the first fraction for the moment, we'll work on the second one.
2vcosθv2sin2θ(1+2hgv2sin2θ)2g\dfrac{2v \cos \theta \sqrt{v^2 \sin^2 \theta (1 + \dfrac{2hg}{v^2 \sin^2 \theta})}}{2g}

Unparseable latex formula:

\dfrac{2v^2 \cos \theta \sin \theta}{2g} \times \sqrt{1 + \dfrac{2hg}{v^2 \sin^2 \theta}


We can use our old friend Mr. Double Angle Formula to simplify it:
v2sin2θ2g1+2hgv2sin2θ\dfrac{v^2 \sin 2\theta}{2g} \sqrt{1 + \dfrac{2hg}{v^2 \sin^2 \theta}}
The whole fraction looks like:
Unparseable latex formula:

R = (\dfrac{v^2\sin 2\theta}{2g}) + (\dfrac{v^2 \sin 2\theta}{2g}) \sqrt{1 + \dfrac{2hg}{v^2 \sin^2 \theta}


And lo and behold, there's a common factor to both fractions. Let's take it out:
R=v22gsin2θ(1+1+2hgv2sin2θ)R = \dfrac{v^2}{2g}\sin 2\theta(1 + \sqrt{1 + \dfrac{2hg}{v^2 \sin^2 \theta}})
And that's what we're looking for.
*I'll finish off the last small bit in a few minutes*
Reply 8
STEP I Question 6

y=f(x)y = f(x)
dy/dx=f(x)dy/dx = f'(x)

The normal to the curve will therefore have gradient -1/f'(x). The equation of the normal would be

yf(x)=1f(x)(xx)y - f(x) = \frac{-1}{f'(x)}(x' - x) (where x' is the general x coordinate of the normal).

At the point Q, x' = 0 as it cuts the y axis.

yf(x)=xf(x)y - f(x) = \frac{x}{f'(x)}

y=xf(x)+f(x)y = \frac{x}{f'(x)} + f(x)

The distance PQ can be worked out using Pythagoras' theorem.

PQ2=(f(x)(xf(x)+f(x)))2+x2PQ^2 = (f(x) - (\frac{x}{f'(x)} + f(x)))^2 + x^2

=x2f(x)2+x2= \frac{x^2}{f'(x)^2} + x^2

It's given that PQ2=ex2+x2PQ^2 = e^{x^2} + x^2

So

x2f(x)2+x2=ex2+x2\frac{x^2}{f'(x)^2} + x^2 = e^{x^2} + x^2

x2f(x)2=ex2\frac{x^2}{f'(x)^2} = e^{x^2}

x2ex2=f(x)2\frac{x^2}{e^{x^2}} = f'(x)^2

xe0.5x2=f(x)\frac{x}{e^{0.5x^2}} = f'(x)

1dy=xe0.5x2dx\int 1 \mathrm{d}y = \int \frac{x}{e^{0.5x^2}} \mathrm{d}x

y=xe0.5x2dxy = \int xe^{-0.5x^2} \mathrm{d}x

y=e0.5x2+cy = -e^{-0.5x^2} + c (use a substitution of u = x^2 if you can't see why)

2=1+c-2 = -1 + c

c=1c = -1

y=e0.5x21y = -e^{-0.5x^2} - 1
Reply 9
STEP III, Q8

d2xdt2=4dxdt4dydt\frac{\mathrm{d}^{2}x}{\mathrm{d}t^{2}} = 4\frac{\mathrm{d}x}{\mathrm{d}t} - 4\frac{\mathrm{d}y}{\mathrm{d}t}

d2xdt24dxdt+4x=48e2t+48e2t\frac{\mathrm{d}^{2}x}{\mathrm{d}t^{2}} - 4\frac{\mathrm{d}x}{\mathrm{d}t} + 4x = 48e^{2t} + 48e^{-2t}

Let u=dxdt2xu = \frac{\mathrm{d}x}{\mathrm{d}t} - 2x

Then dudt2u=48e2t+48e2t\frac{\mathrm{d}u}{\mathrm{d}t} - 2u = 48e^{2t} + 48e^{-2t}

Using an integrating factor, e2te^{-2t}, we get

ue2t=48+48e4t  dt=48t12e4t+cue^{-2t} = \int 48 + 48e^{-4t} \; \mathrm{d}t = 48t - 12e^{-4t} + c

u=e2t(48t+c)12e2tu = e^{2t}(48t + c) - 12e^{-2t}

dxdt2x=e2t(48t+c)12e2t\frac{\mathrm{d}x}{\mathrm{d}t} - 2x = e^{2t}(48t + c) - 12e^{-2t}

xe2t=48t+c12e4t  dt=24t2+ct+3e4t+dxe^{-2t} = \int 48t + c - 12e^{-4t} \; \mathrm{d}t = 24t^{2} + ct + 3e^{-4t} + d

x=e2t(24t2+ct+d)+3e2tx = e^{2t}(24t^{2} + ct + d) + 3e^{-2t}

Applying boundary conditions:

0=d+3d=30 = d + 3 \Rightarrow d = -3

dxdt=2e2t(24t2+ct+d)+e2t(48t+c)6e2t=4(xy)\frac{\mathrm{d}x}{\mathrm{d}t} = 2e^{2t}(24t^{2} + ct + d) + e^{2t}(48t + c) - 6e^{-2t} = 4(x-y)

2d+c6=0c=62d=6+6=122d + c - 6 = 0 \Rightarrow c = 6 - 2d = 6 + 6 = 12

x=e2t(24t2+12t3)+3e2tx = e^{2t}(24t^{2} + 12t - 3) + 3e^{-2t}

4y=4xdxdt=e2t(48t24t18)+18e2t=4te2t(12t1)36sinh2t4y = 4x - \frac{\mathrm{d}x}{\mathrm{d}t} = e^{2t}(48t^{2} - 4t - 18) + 18e^{-2t} = 4te^{2t}(12t - 1) - 36\sinh 2t

y=te2t(12t1)9sinh2ty = te^{2t}(12t - 1) - 9\sinh 2t

---

I think that I shall restrict myself to a maximum of three solutions per day (at least at the start), so that everyone has a chance to answer some if they want.
Reply 10
Hang on a sec, mine was mislabelled, should be Step I not step II.. sorry about that!

edit: thanks.. now typing up II/2 :p:
Reply 11
STEP II/2

tanx=0anxn\displaystyle \tan x = \sum^{\infty}_0 a_n x^n


xcotx=1+1bnxn\displaystyle x \cot x = 1 + \sum^{\infty}_1 b_n x^n

()cotx=1x+1bnxn1\displaystyle (*) \cot x = \frac{1}{x} + \sum^{\infty}_1 b_n x^{n-1}

=1x+0bn+1xn\displaystyle = \frac{1}{x} + \sum^{\infty}_0 b_{n+1} x^n



2cot2x=cotxtanx\displaystyle 2 \cot 2x = \cot x - \tan x

Therefore, by (*),

2cot2x=1x+0bn+1xn0anxn\displaystyle 2 \cot 2x = \frac{1}{x} + \sum^{\infty}_0 b_{n+1} x^n - \sum^{\infty}_0 a_n x^n

=1x+0(bn+1an)xn\displaystyle = \frac{1}{x} + \sum^{\infty}_0 (b_{n+1} - a_n) x^n


we can also use (*) to express 2cot2x as 2(12x+0bn+1(2x)n)=1x+20bn+12nxn\displaystyle 2(\frac{1}{2x} + \sum^{\infty}_0 b_{n+1} (2x)^n) = \frac{1}{x} + 2\sum^{\infty}_0 b_{n+1} 2^n x^n


Therefore,

1x+20bn+12nxn=1x+0(bn+1an)xn\displaystyle \frac{1}{x} + 2\sum^{\infty}_0 b_{n+1} 2^n x^n = \frac{1}{x} + \sum^{\infty}_0 (b_{n+1} - a_n) x^n

0bn+12n+1xn=0(bn+1an)xn\displaystyle \sum^{\infty}_0 b_{n+1} 2^{n+1} x^n = \sum^{\infty}_0 (b_{n+1} - a_n) x^n

we can compare the coefficients of the respective summations and say

bn+12n+1=bn+1an\displaystyle b_{n+1} 2^{n+1} = b_{n+1} - a_n

an=bn+1bn+12n+1\displaystyle a_n = b_{n+1} - b_{n+1} 2^{n+1}

an1=bn(12n)\displaystyle a_{n-1} = b_n(1 - 2^n)


for the second part, note that

xcscx=1+1cnxn\displaystyle x \csc x = 1 + \sum^{\infty}_1 c_n x^n

cscx=1x+1cnxn1\displaystyle \csc x = \frac{1}{x} + \sum^{\infty}_1 c_n x^{n-1}

=1x+0cn+1xn\displaystyle = \frac{1}{x} + \sum^{\infty}_0 c_{n+1} x^n


we have to find an identity for 2csc2x

2csc2x=2sin2x=22sinxcosx=1sinxcosx\displaystyle 2 \csc 2x = \frac{2}{\sin 2x} = \frac{2}{2 \sin x \cos x} = \frac{1}{\sin x \cos x}

(using partial fractions)
=sin2x+cos2xsinxcosx=cosxsinx+sinxcosx=cotx+tanx\displaystyle = \frac{\sin^2 x + cos^2 x}{\sin x \cos x} = \frac{\cos x}{\sin x} + \frac{\sin x}{\cos x} = \cot x + \tan x


Also,
2csc2x=2(12x+0cn+1(2x)n)\displaystyle 2 \csc 2x = 2(\frac{1}{2x} + \sum^{\infty}_0 c_{n+1} (2x)^n)

=2(12x+0cn+12nxn)\displaystyle = 2(\frac{1}{2x} + \sum^{\infty}_0 c_{n+1} 2^n x^n)

=1x+0cn+12n+1xn\displaystyle = \frac{1}{x} + \sum^{\infty}_0 c_{n+1} 2^{n+1} x^n



cotx+tanx=1x+0bn+1xn+0anxn\displaystyle \cot x + \tan x = \frac{1}{x} + \sum^{\infty}_0 b_{n+1} x^n + \sum^{\infty}_0 a_n x^n

2csc2x=1x+0bn+1xn+0anxn\displaystyle 2 \csc 2x = \frac{1}{x} + \sum^{\infty}_0 b_{n+1} x^n + \sum^{\infty}_0 a_n x^n


2csc2x=cotx+tanx2 \csc 2x = \cot x + \tan x, so:


1x+0cn+12n+1xn=1x+0bn+1xn+0anxn\displaystyle \frac{1}{x} + \sum^{\infty}_0 c_{n+1} 2^{n+1} x^n = \frac{1}{x} + \sum^{\infty}_0 b_{n+1} x^n + \sum^{\infty}_0 a_n x^n

0cn+12n+1xn=0bn+1xn+0anxn\displaystyle \sum^{\infty}_0 c_{n+1} 2^{n+1} x^n = \sum^{\infty}_0 b_{n+1} x^n + \sum^{\infty}_0 a_n x^n

0cn+12n+1xn=0(bn+1+an)xn\displaystyle \sum^{\infty}_0 c_{n+1} 2^{n+1} x^n = \sum^{\infty}_0 (b_{n+1} + a_n) x^n


Comparing coefficients of the polynomials (in x) of the expansion,

cn+12n+1=bn+1+an\displaystyle c_{n+1} 2^{n+1} = b_{n+1} + a_n

Rearranging and substituting in the earlier definition of an1=bn(12n)an112n=bna_{n-1} = b_n(1-2^n) \Longrightarrow \frac{a_{n-1}}{1-2^n} = b_n,


cn+12n+1=an12n+1+an\displaystyle c_{n+1} 2^{n+1} = \frac{a_{n}}{1-2^{n+1}} + a_n

cn+1=(112n+1+1)(12n+1)an\displaystyle c_{n+1} = (\frac{1}{1 - 2^{n+1}}+1)(\frac{1}{2^{n+1}})a_n

cn+1=(22n+112n+1)(12n+1)an\displaystyle c_{n+1} = (\frac{2 - 2^{n+1}}{1 - 2^{n+1}})(\frac{1}{2^{n+1}})a_n

cn+1=(12n12n+1)(12n)an\displaystyle c_{n+1} = (\frac{1 - 2^n}{1 - 2^{n+1}})(\frac{1}{2^n})a_n

cn+1=(2n12n+11)(12n)an\displaystyle c_{n+1} = (\frac{2^n - 1}{2^{n+1} - 1})(\frac{1}{2^n})a_n

cn+1=(2n112n1)(12n1)an1\displaystyle c_{n+1} = (\frac{2^{n-1} - 1}{2^{n} - 1})(\frac{1}{2^{n-1}})a_{n-1}

any corrections welcome !
Reply 12
STEP II, (11) attached

attachments 2&3 are in the wrong order
Reply 14
STEP II (6) attached
Reply 15
STEP II Question 3

2(u+iv)=ex+iye(x+iy)=2sinh(x+iy)=2sinh(x)cosh(iy)+2cosh(x)sinh(yi)=2sinh(x)cos(y)+2isin(y)cosh(x) 2(u + iv) = e^{x+iy} - e^{-(x+iy)} = 2\sinh(x + iy) = 2\sinh(x)\cosh(iy) + 2\cosh(x)\sinh(yi) = 2\sinh(x)\cos(y) + 2i\sin(y)\cosh(x)

Equating real and imaginary parts:

u=sinh(x)cos(y)u = \sinh(x)\cos(y) (*)

v=sin(y)cosh(x)v = \sin(y)\cosh(x) (**)

if x=a, then:

u=sinh(a)cos(y)u = \sinh(a)\cos(y) (1)

v=sin(y)cosh(a)v = \sin(y)\cosh(a) (2)

Now, subbing (1) and (2) into:

cos2y+sin2y=1 \cos^{2}y + \sin^{2}y = 1

We get:

(usinh(a))2+(vcosh(a))2=1() (\frac{u}{\sinh(a)})^{2} + (\frac{v}{\cosh(a)})^{2} = 1 (\triangle)

As required.

Substituting y=b into (*) and (**):

u=sinh(x)cos(b)u = \sinh(x)\cos(b) (3)

v=cosh(x)sin(b)v = \cosh(x)\sin(b) (4).

This time, using cosh2xsinh2x=1 \cosh^{2}x - \sinh^{2}x = 1, we end up with:

(vsin(b))2(ucos(b))2=1() (\frac{v}{\sin(b)})^{2} - (\frac{u}{\cos(b)})^{2} = 1 (\heartsuit) .

Since 0<sin(b)<10 < \sin(b) < 1 and cosh(x)1 \cosh(x) \geq 1, v>0\Rightarrow v>0. Hence, (\heartsuit) corresponds to a single hyeprbola branch, which is above the line v=0.

One point of intersection is: u=sinh(a)cos(b) u = \sinh(a)\cos(b) and v=sin(b)cosh(a) v = \sin(b)\cosh(a) .


Differentiating \triangle implicitly we get:

dvdu=(uv)coth2(a) \frac{dv}{du} = (\frac{u}{v})\coth^{2}(a) .

Which is equal to: cot(b)coth(a) -\cot(b)\coth(a) at u=sinh(a)cos(b)u = \sinh(a)\cos(b) and v=sin(b)cosh(a) v = \sin(b)\cosh(a).

Differentiating \heartsuit implicitly we get:

dvdu=(uv)tan2(b)\frac{dv}{du} = (\frac{u}{v})\tan^{2}(b) .

Which is, tanh(a)tan(b)\tanh(a)\tan(b) when u=sinh(a)cos(b)u = \sinh(a)\cos(b) and v=sin(b)cosh(a) v = \sin(b)\cosh(a).

tanh(a)tan(b)\tanh(a)\tan(b)Xcot(b)coth(a)=1-\cot(b)\coth(a) = -1

Hence the curves intersect and right angles at this point.

Regarding the sketch, I think the elipse becomes a vertical straight line from v = -1 to v = 1, and the hyperbola becomes the line v = 1.
Reply 16
STEP 1, Q3

x=xa,  y=(1x)b\textbf{x} = x\textbf{a}, \; \textbf{y} = (1-x)\textbf{b}

c=13(2a+b),d=13(a+2b)\textbf{c} = \frac{1}{3}(2\textbf{a} + \textbf{b}), \textbf{d} = \frac{1}{3}(\textbf{a} + 2\textbf{b})

(By the ratio theorem.)

CY=(23x)b23a\overrightarrow{CY} = (\frac{2}{3} - x)\textbf{b} - \frac{2}{3}\textbf{a}
DX=(x13)a23b\overrightarrow{DX} = (x - \frac{1}{3})\textbf{a} - \frac{2}{3}\textbf{b}

These vectors are perpendicular, so the scalar product is zero.

(29+xx2)a.b29=0(\frac{2}{9} + x - x^{2})\textbf{a}.\textbf{b} - \frac{2}{9} = 0

(Note that the scalar product is commutative and distributive.)

a.b=29x29x2\textbf{a}.\textbf{b} = \frac{-2}{9x^{2} - 9x - 2}

Let f(x)=29x29x2,  0x1f(x) = \frac{-2}{9x^{2} - 9x - 2}, \; 0 \leq x \leq 1

f(x)=2(18x9)(9x29x2)2f'(x) = \frac{2(18x - 9)}{(9x^{2}-9x-2)^{2}}

Minimum and maximum values occur at the endpoint of a range or at a turning point. There is a turning point when x = 1/2.

f(0)=1,  f(12)=817,  f(1)=1f(0) = 1, \; f(\frac{1}{2}) = \frac{8}{17}, \; f(1) = 1

So 817a.b1\frac{8}{17} \leq \textbf{a}.\textbf{b} \leq 1

Note that a.b=abcosθ=cosθ\textbf{a}.\textbf{b} = |a||b|\cos\theta = \cos\theta

Where θ=AOB,  0<θ<π\theta = \angle AOB, \; 0 < \theta < \pi

The maximum value is θ\theta occurs when cosθ=817\cos\theta = \frac{8}{17}

Suppose that λCY=CE,  μDX=DE\lambda \overrightarrow{CY} = \overrightarrow{CE},\; \mu \overrightarrow{DX} = \overrightarrow{DE}

Then λ(16b23a)=13(ba)+μ(16a23b)\lambda (\frac{1}{6} \textbf{b} - \frac{2}{3} \textbf{a}) = \frac{1}{3} ( \textbf{b} - \textbf{a} ) + \mu ( \frac{1}{6} \textbf{a} - \frac{2}{3} \textbf{b} )

So 16λ=1323μ\frac{1}{6} \lambda = \frac{1}{3} - \frac{2}{3}\mu

23λ=13+16μ-\frac{2}{3}\lambda = -\frac{1}{3} + \frac{1}{6}\mu

λ=μ=25\lambda = \mu = \frac{2}{5}

e=c+25CY=25a+25b\textbf{e} = \textbf{c} + \frac{2}{5}\overrightarrow{CY} = \frac{2}{5}\textbf{a} + \frac{2}{5}\textbf{b}

By the cosine rule, AB2=1817AB^{2} = \frac{18}{17}
Let F be the midpoint of AB. By Pythagoras, OF2=1934=2534,  OF=534OF^{2} = 1 - \frac{9}{34} = \frac{25}{34}, \; OF = \frac{5}{\sqrt{34}}

OE=35×534=334OE = \frac{3}{5} \times \frac{5}{\sqrt{34}} = \frac{3}{\sqrt{34}}
Reply 17
*bump* (just to remind people that this thread is still here :biggrin:) - I have solutions that I can type up if no one else wants to, but I really don't want to take over the thread..
Reply 18
STEP I - Question 7

*Graph Attached*

If we work out the area of a quarter of the rectangle, which is in the top right hand bit of the axes, and maximise this, it make calculation soooo much eaiser. (Since we can forget about modulus signs and y being negative) :

Let the area of the rectangle be AA.

Base of smaller rectangle = 1-x
height = (1(1x))=x\sqrt(1-(1-x)) = \sqrt x

so 14A=(1x)x\frac{1}{4}A = (1-x)\sqrt x

which we want to maximise.

d14Adx=13x2x\frac{d\frac{1}{4}A}{dx} = \frac{1-3x}{2\sqrt x}

Which is zero when x=13 x = \frac{1}{3}

Finding the second derivative and subsituting x = 13\frac{1}{3} in shows 14A\frac{1}{4}A is a maximum at this point.

So Amax=4×[(113)13]=827A_{max} = 4 \times [(1-\frac{1}{3})\frac{1}{\sqrt 3}] = \frac{8}{\sqrt 27} .

________

We can do the second bit in the same way, since its a closed curve that is symmetrical in both axes.

Let the area of the second rectangle be BB.
Working in the top right quadrant again;
Base = 1-x
height = (1(1x))n2m=xn2m(1-(1-x))^{\frac{n}{2m}} = x^{\frac{n}{2m}}.

So 14B=xn2m(1x)\frac{1}{4}B = x^{\frac{n}{2m}}(1-x) .

d14Bdx=xn2m1(n2mx(1+n2m)) \frac{d\frac{1}{4}B}{dx} = x^{\frac{n}{2m} -1}(\frac{n}{2m} - x(1 + \frac{n}{2m}))

Which is zero when x = 0 or x = n2m+n\frac{n}{2m + n} .

Clearly the area is a minimum when x= 0, and so is a maximum when x =n2m+n\frac{n}{2m + n} .

Bmax=4×[(n2m+n)n2m(1n2m+n)]=4(n2m+n)n2m(2m2m+n)\Rightarrow B_{max} = 4 \times [(\frac{n}{2m+n})^{\frac{n}{2m}}(1 - \frac{n}{2m +n})] = 4(\frac{n}{2m + n})^{\frac{n}{2m}}(\frac{2m}{2m+n}) .
I've got a solution to I question 9 , but don't have a scanner available so will add graphs when I get the opportunity.

Question 9 STEP I

y=x3832xy=\frac{x^3}{8}-\frac{3}{2}x and y=38x232y'=\frac{3}{8}x^2-\frac{3}{2} Setting y'=0 x2=4x=±2\Rightarrow x^2=4 \Rightarrow x=\pm2
f(2)=-2, f(-2)=2, also for graphing purposes it may be worth to note that f(0)=0 and f(4)=2 and f(-4)=-2. y=0 has solutions x=0 or x=±12=±23x=\pm\sqrt{12}=\pm2\sqrt{3} f''(-2)=- i.e. local max f''(2)=+ i.e. local min
Graph, see attachment

(a)
Unparseable latex formula:

X=\frac{1}{2}x\,\Y=y

i.e. 2X=x2X=x therefore the equation of y in this X-Y plane is Y=(2X)38322X=X33X=X(X23)Y=\frac{(2X)^3}{8}-\frac{3}{2}2X=X^3-3X=X(X^2-3)
Y=3X23=3(x21)Y'=3X^2-3=3(x^2-1) setting it equal to 0 gives x=±1x=\pm1 f(1)=-2 ans f(-1)=2
For graphing it can also be good to see that f(2)=2, f(-2)=-2 and y=0x=0,x=±3y=0 \Rightarrow x=0, x=\pm\sqrt{3}
Graph, see attachment

(b) X=x,Y=12yX=x, Y=\frac{1}{2}y i.e. 2Y=y2Y=y therefore the equation of y in this X-Y plane is Y=X31634X=X4(X243)Y=\frac{X^3}{16}-\frac{3}{4}X=\frac{X}{4}(\frac{X^2}{4}-3)
Y=316X234=34(X241)Y'=\frac{3}{16}X^2-\frac{3}{4}=\frac{3}{4}(\frac{X^2}{4}-1) Setting this equal to 0 gives X=±2X=\pm2. f(2)=-1 and f(-2)=1
For graphing, also see that Y=0x=0,x=±23Y=0 \Rightarrow x=0, x= \pm2\sqrt{3}, f(4)=1 and f(-4)=-1
Graph, see attachment.

(c) X=12x+1,Y=yX=\frac{1}{2}x+1, Y=y i.e. x=2(X1)x=2(X-1) therefore the equation in the X-Y plane is Y=(2(X1))38322(X1)=(X1)33(X1)=(X1)(X22X2)Y=\frac{(2(X-1))^3}{8} - \frac{3}{2}2(X-1)=(X-1)^3 - 3(X-1) = (X-1)(X^2-2X-2)
Y=3(X1)23=3((X1)21)=3(X22X)=3X(X2)Y'=3(X-1)^2-3=3((X-1)^2-1)=3(X^2-2X)=3X(X-2) Setting this equal to 0 gives x=0,x=2 x=0, x=2 f(0)=2, f(2)=-2.
For graphing, also see that Y=0X=1,X=1±3Y=0 \Rightarrow X=1, X=1\pm\sqrt{3} and f(-1)=-2, f(3)=2

(d) X=x,Y=12y+1X=x, Y=\frac{1}{2}y+1 i.e. y=2(Y1)y=2(Y-1) therefore the equation in the X-Y plane is Y=X31634X+1Y=\frac{X^3}{16}-\frac{3}{4}X+1
Y=316X234=34(X241)Y'=\frac{3}{16}X^2-\frac{3}{4}=\frac{3}{4}(\frac{X^2}{4}-1) setting this equal to 0 gives X=±2 X=\pm2 f(2)=0 f(-2)=2
For graphing I'll check f(4) and f(-4) too, which give 2 and 0 respectively.

For the last part, we're looking for a graph in the X-Y plane with local min (0,0) and local max (1,1). To obtain this on the form X=ax+b and Y=cy+d, first observe that for a graph of a cubic, changing the sign means a reflection in the x-axis i.e. local max and min swap places, this is what we need to do. That means either a or c is negative (but not both!). a squeezes the graph along the X-axis and c squeezes it along the Y-axis, c moves the graph to the left/right and d moves the graph up/down. Knowing these we can see that X=14x+12X=-\frac{1}{4}x+\frac{1}{2} and Y=14y+12Y=\frac{1}{4}y+\frac{1}{2} will produce the desired graph.

Okay, so I hope I've interpreted everything correct in the question. Seems pretty nice as a question.

Quick Reply

Latest